Reducing system of 10 inequalities












1












$begingroup$


I have the following system of inequalities of real variables



sys = 0.383706 x1 + 0.720204 x2 + 1.4568 x3 - 0.000244875 y >   0 && -0.0941312 x1 - 0.176681 x2 - 0.357383 x3 + 0.0000592689 y >   0 && 1.41819 x1 + 3.079 x2 + 2.53801 x3 - 0.00182772 y >   0 && -0.00258971 x1 - 0.00562247 x2 - 0.00463458 x3 +    3.93843*10^-6 y > 0 &&  0.129511 x1 + 0.214698 x2 + 0.286739 x3 - 0.0000795243 y >   0 && -0.660462 x1 - 1.09489 x2 - 1.46227 x3 + 0.000405426 y > 0 &&  0.351966 x1 + 3.44019 x2 + 7.59863 x3 + 0.00257072 y >   0 && -0.00265509 x1 - 0.0259514 x2 - 0.0573209 x3 -    0.0000199557 y > 0 &&  1.34471 x1 + 2.58639 x2 + 3.39561 x3 + 0.0000742173 y >   0 && -0.0314846 x1 - 0.0605571 x2 - 0.079504 x3 - 1.69328*10^-6 y >   0


I want to Reduce the system in order to see if it admits solutions or not. However, the command Reduce takes too much time and gets stuck. How can you check if this system admits solutions?



I have tried with FindInstance[sys,{x1,x2,x3,y},Reals] which returns {}. However, I am not sure this is a fully proof that the system does not admit solution. Indeed, If I run the command on a subsystem of inequalities I experience the following behaviour



FindInstance[sys[[1 ;; 4]], {x1, x2, x3, y}, Reals]
(* {} *)


but If I ask for more instances, Mathematica found the points



FindInstance[sys[[1 ;; 4]], {x1, x2, x3, y}, Reals, 2]
(*{{x1 -> -33., x2 -> 13.5619, x3 -> 1.98727, y -> 0.255564}, {x1 -> -51., x2 -> 20.9595, x3 -> 3.07117, y -> 0.601467}}*)


So, the subsystem [[1;;4]] admits at least a solution. I have run FindInstance[sys, {x1, x2, x3, y}, Reals, 2] for all the night and got the output {}. Still, is this a proof that the system does not admits solution?










share|improve this question









$endgroup$








  • 1




    $begingroup$
    Reduce[Rationalize[sys, 2^-12], {x1, x2, x3, y}, Reals] gives False.
    $endgroup$
    – kglr
    Nov 19 '18 at 7:28






  • 1




    $begingroup$
    and RegionIntersection @@ (ImplicitRegion[#, {x1, x2, x3, y}] & /@ (List @@ sys)) gives EmptyRegion[4].
    $endgroup$
    – kglr
    Nov 19 '18 at 7:33
















1












$begingroup$


I have the following system of inequalities of real variables



sys = 0.383706 x1 + 0.720204 x2 + 1.4568 x3 - 0.000244875 y >   0 && -0.0941312 x1 - 0.176681 x2 - 0.357383 x3 + 0.0000592689 y >   0 && 1.41819 x1 + 3.079 x2 + 2.53801 x3 - 0.00182772 y >   0 && -0.00258971 x1 - 0.00562247 x2 - 0.00463458 x3 +    3.93843*10^-6 y > 0 &&  0.129511 x1 + 0.214698 x2 + 0.286739 x3 - 0.0000795243 y >   0 && -0.660462 x1 - 1.09489 x2 - 1.46227 x3 + 0.000405426 y > 0 &&  0.351966 x1 + 3.44019 x2 + 7.59863 x3 + 0.00257072 y >   0 && -0.00265509 x1 - 0.0259514 x2 - 0.0573209 x3 -    0.0000199557 y > 0 &&  1.34471 x1 + 2.58639 x2 + 3.39561 x3 + 0.0000742173 y >   0 && -0.0314846 x1 - 0.0605571 x2 - 0.079504 x3 - 1.69328*10^-6 y >   0


I want to Reduce the system in order to see if it admits solutions or not. However, the command Reduce takes too much time and gets stuck. How can you check if this system admits solutions?



I have tried with FindInstance[sys,{x1,x2,x3,y},Reals] which returns {}. However, I am not sure this is a fully proof that the system does not admit solution. Indeed, If I run the command on a subsystem of inequalities I experience the following behaviour



FindInstance[sys[[1 ;; 4]], {x1, x2, x3, y}, Reals]
(* {} *)


but If I ask for more instances, Mathematica found the points



FindInstance[sys[[1 ;; 4]], {x1, x2, x3, y}, Reals, 2]
(*{{x1 -> -33., x2 -> 13.5619, x3 -> 1.98727, y -> 0.255564}, {x1 -> -51., x2 -> 20.9595, x3 -> 3.07117, y -> 0.601467}}*)


So, the subsystem [[1;;4]] admits at least a solution. I have run FindInstance[sys, {x1, x2, x3, y}, Reals, 2] for all the night and got the output {}. Still, is this a proof that the system does not admits solution?










share|improve this question









$endgroup$








  • 1




    $begingroup$
    Reduce[Rationalize[sys, 2^-12], {x1, x2, x3, y}, Reals] gives False.
    $endgroup$
    – kglr
    Nov 19 '18 at 7:28






  • 1




    $begingroup$
    and RegionIntersection @@ (ImplicitRegion[#, {x1, x2, x3, y}] & /@ (List @@ sys)) gives EmptyRegion[4].
    $endgroup$
    – kglr
    Nov 19 '18 at 7:33














1












1








1





$begingroup$


I have the following system of inequalities of real variables



sys = 0.383706 x1 + 0.720204 x2 + 1.4568 x3 - 0.000244875 y >   0 && -0.0941312 x1 - 0.176681 x2 - 0.357383 x3 + 0.0000592689 y >   0 && 1.41819 x1 + 3.079 x2 + 2.53801 x3 - 0.00182772 y >   0 && -0.00258971 x1 - 0.00562247 x2 - 0.00463458 x3 +    3.93843*10^-6 y > 0 &&  0.129511 x1 + 0.214698 x2 + 0.286739 x3 - 0.0000795243 y >   0 && -0.660462 x1 - 1.09489 x2 - 1.46227 x3 + 0.000405426 y > 0 &&  0.351966 x1 + 3.44019 x2 + 7.59863 x3 + 0.00257072 y >   0 && -0.00265509 x1 - 0.0259514 x2 - 0.0573209 x3 -    0.0000199557 y > 0 &&  1.34471 x1 + 2.58639 x2 + 3.39561 x3 + 0.0000742173 y >   0 && -0.0314846 x1 - 0.0605571 x2 - 0.079504 x3 - 1.69328*10^-6 y >   0


I want to Reduce the system in order to see if it admits solutions or not. However, the command Reduce takes too much time and gets stuck. How can you check if this system admits solutions?



I have tried with FindInstance[sys,{x1,x2,x3,y},Reals] which returns {}. However, I am not sure this is a fully proof that the system does not admit solution. Indeed, If I run the command on a subsystem of inequalities I experience the following behaviour



FindInstance[sys[[1 ;; 4]], {x1, x2, x3, y}, Reals]
(* {} *)


but If I ask for more instances, Mathematica found the points



FindInstance[sys[[1 ;; 4]], {x1, x2, x3, y}, Reals, 2]
(*{{x1 -> -33., x2 -> 13.5619, x3 -> 1.98727, y -> 0.255564}, {x1 -> -51., x2 -> 20.9595, x3 -> 3.07117, y -> 0.601467}}*)


So, the subsystem [[1;;4]] admits at least a solution. I have run FindInstance[sys, {x1, x2, x3, y}, Reals, 2] for all the night and got the output {}. Still, is this a proof that the system does not admits solution?










share|improve this question









$endgroup$




I have the following system of inequalities of real variables



sys = 0.383706 x1 + 0.720204 x2 + 1.4568 x3 - 0.000244875 y >   0 && -0.0941312 x1 - 0.176681 x2 - 0.357383 x3 + 0.0000592689 y >   0 && 1.41819 x1 + 3.079 x2 + 2.53801 x3 - 0.00182772 y >   0 && -0.00258971 x1 - 0.00562247 x2 - 0.00463458 x3 +    3.93843*10^-6 y > 0 &&  0.129511 x1 + 0.214698 x2 + 0.286739 x3 - 0.0000795243 y >   0 && -0.660462 x1 - 1.09489 x2 - 1.46227 x3 + 0.000405426 y > 0 &&  0.351966 x1 + 3.44019 x2 + 7.59863 x3 + 0.00257072 y >   0 && -0.00265509 x1 - 0.0259514 x2 - 0.0573209 x3 -    0.0000199557 y > 0 &&  1.34471 x1 + 2.58639 x2 + 3.39561 x3 + 0.0000742173 y >   0 && -0.0314846 x1 - 0.0605571 x2 - 0.079504 x3 - 1.69328*10^-6 y >   0


I want to Reduce the system in order to see if it admits solutions or not. However, the command Reduce takes too much time and gets stuck. How can you check if this system admits solutions?



I have tried with FindInstance[sys,{x1,x2,x3,y},Reals] which returns {}. However, I am not sure this is a fully proof that the system does not admit solution. Indeed, If I run the command on a subsystem of inequalities I experience the following behaviour



FindInstance[sys[[1 ;; 4]], {x1, x2, x3, y}, Reals]
(* {} *)


but If I ask for more instances, Mathematica found the points



FindInstance[sys[[1 ;; 4]], {x1, x2, x3, y}, Reals, 2]
(*{{x1 -> -33., x2 -> 13.5619, x3 -> 1.98727, y -> 0.255564}, {x1 -> -51., x2 -> 20.9595, x3 -> 3.07117, y -> 0.601467}}*)


So, the subsystem [[1;;4]] admits at least a solution. I have run FindInstance[sys, {x1, x2, x3, y}, Reals, 2] for all the night and got the output {}. Still, is this a proof that the system does not admits solution?







equation-solving inequalities findinstance






share|improve this question













share|improve this question











share|improve this question




share|improve this question










asked Nov 19 '18 at 7:18









apt45apt45

561211




561211








  • 1




    $begingroup$
    Reduce[Rationalize[sys, 2^-12], {x1, x2, x3, y}, Reals] gives False.
    $endgroup$
    – kglr
    Nov 19 '18 at 7:28






  • 1




    $begingroup$
    and RegionIntersection @@ (ImplicitRegion[#, {x1, x2, x3, y}] & /@ (List @@ sys)) gives EmptyRegion[4].
    $endgroup$
    – kglr
    Nov 19 '18 at 7:33














  • 1




    $begingroup$
    Reduce[Rationalize[sys, 2^-12], {x1, x2, x3, y}, Reals] gives False.
    $endgroup$
    – kglr
    Nov 19 '18 at 7:28






  • 1




    $begingroup$
    and RegionIntersection @@ (ImplicitRegion[#, {x1, x2, x3, y}] & /@ (List @@ sys)) gives EmptyRegion[4].
    $endgroup$
    – kglr
    Nov 19 '18 at 7:33








1




1




$begingroup$
Reduce[Rationalize[sys, 2^-12], {x1, x2, x3, y}, Reals] gives False.
$endgroup$
– kglr
Nov 19 '18 at 7:28




$begingroup$
Reduce[Rationalize[sys, 2^-12], {x1, x2, x3, y}, Reals] gives False.
$endgroup$
– kglr
Nov 19 '18 at 7:28




1




1




$begingroup$
and RegionIntersection @@ (ImplicitRegion[#, {x1, x2, x3, y}] & /@ (List @@ sys)) gives EmptyRegion[4].
$endgroup$
– kglr
Nov 19 '18 at 7:33




$begingroup$
and RegionIntersection @@ (ImplicitRegion[#, {x1, x2, x3, y}] & /@ (List @@ sys)) gives EmptyRegion[4].
$endgroup$
– kglr
Nov 19 '18 at 7:33










1 Answer
1






active

oldest

votes


















3












$begingroup$

Reduce[Rationalize[sys, 2^-12], {x1, x2, x3, y}, Reals] // RepeatedTiming



{0.24878, False}




RegionIntersection @@ (ImplicitRegion[#, {x1, x2, x3, y}] & /@ (List @@ sys)) // 
RepeatedTiming



{0.0152, EmptyRegion[4]}







share|improve this answer









$endgroup$













    Your Answer





    StackExchange.ifUsing("editor", function () {
    return StackExchange.using("mathjaxEditing", function () {
    StackExchange.MarkdownEditor.creationCallbacks.add(function (editor, postfix) {
    StackExchange.mathjaxEditing.prepareWmdForMathJax(editor, postfix, [["$", "$"], ["\\(","\\)"]]);
    });
    });
    }, "mathjax-editing");

    StackExchange.ready(function() {
    var channelOptions = {
    tags: "".split(" "),
    id: "387"
    };
    initTagRenderer("".split(" "), "".split(" "), channelOptions);

    StackExchange.using("externalEditor", function() {
    // Have to fire editor after snippets, if snippets enabled
    if (StackExchange.settings.snippets.snippetsEnabled) {
    StackExchange.using("snippets", function() {
    createEditor();
    });
    }
    else {
    createEditor();
    }
    });

    function createEditor() {
    StackExchange.prepareEditor({
    heartbeatType: 'answer',
    autoActivateHeartbeat: false,
    convertImagesToLinks: false,
    noModals: true,
    showLowRepImageUploadWarning: true,
    reputationToPostImages: null,
    bindNavPrevention: true,
    postfix: "",
    imageUploader: {
    brandingHtml: "Powered by u003ca class="icon-imgur-white" href="https://imgur.com/"u003eu003c/au003e",
    contentPolicyHtml: "User contributions licensed under u003ca href="https://creativecommons.org/licenses/by-sa/3.0/"u003ecc by-sa 3.0 with attribution requiredu003c/au003e u003ca href="https://stackoverflow.com/legal/content-policy"u003e(content policy)u003c/au003e",
    allowUrls: true
    },
    onDemand: true,
    discardSelector: ".discard-answer"
    ,immediatelyShowMarkdownHelp:true
    });


    }
    });














    draft saved

    draft discarded


















    StackExchange.ready(
    function () {
    StackExchange.openid.initPostLogin('.new-post-login', 'https%3a%2f%2fmathematica.stackexchange.com%2fquestions%2f186279%2freducing-system-of-10-inequalities%23new-answer', 'question_page');
    }
    );

    Post as a guest















    Required, but never shown

























    1 Answer
    1






    active

    oldest

    votes








    1 Answer
    1






    active

    oldest

    votes









    active

    oldest

    votes






    active

    oldest

    votes









    3












    $begingroup$

    Reduce[Rationalize[sys, 2^-12], {x1, x2, x3, y}, Reals] // RepeatedTiming



    {0.24878, False}




    RegionIntersection @@ (ImplicitRegion[#, {x1, x2, x3, y}] & /@ (List @@ sys)) // 
    RepeatedTiming



    {0.0152, EmptyRegion[4]}







    share|improve this answer









    $endgroup$


















      3












      $begingroup$

      Reduce[Rationalize[sys, 2^-12], {x1, x2, x3, y}, Reals] // RepeatedTiming



      {0.24878, False}




      RegionIntersection @@ (ImplicitRegion[#, {x1, x2, x3, y}] & /@ (List @@ sys)) // 
      RepeatedTiming



      {0.0152, EmptyRegion[4]}







      share|improve this answer









      $endgroup$
















        3












        3








        3





        $begingroup$

        Reduce[Rationalize[sys, 2^-12], {x1, x2, x3, y}, Reals] // RepeatedTiming



        {0.24878, False}




        RegionIntersection @@ (ImplicitRegion[#, {x1, x2, x3, y}] & /@ (List @@ sys)) // 
        RepeatedTiming



        {0.0152, EmptyRegion[4]}







        share|improve this answer









        $endgroup$



        Reduce[Rationalize[sys, 2^-12], {x1, x2, x3, y}, Reals] // RepeatedTiming



        {0.24878, False}




        RegionIntersection @@ (ImplicitRegion[#, {x1, x2, x3, y}] & /@ (List @@ sys)) // 
        RepeatedTiming



        {0.0152, EmptyRegion[4]}








        share|improve this answer












        share|improve this answer



        share|improve this answer










        answered Nov 19 '18 at 7:59









        kglrkglr

        181k10200413




        181k10200413






























            draft saved

            draft discarded




















































            Thanks for contributing an answer to Mathematica Stack Exchange!


            • Please be sure to answer the question. Provide details and share your research!

            But avoid



            • Asking for help, clarification, or responding to other answers.

            • Making statements based on opinion; back them up with references or personal experience.


            Use MathJax to format equations. MathJax reference.


            To learn more, see our tips on writing great answers.




            draft saved


            draft discarded














            StackExchange.ready(
            function () {
            StackExchange.openid.initPostLogin('.new-post-login', 'https%3a%2f%2fmathematica.stackexchange.com%2fquestions%2f186279%2freducing-system-of-10-inequalities%23new-answer', 'question_page');
            }
            );

            Post as a guest















            Required, but never shown





















































            Required, but never shown














            Required, but never shown












            Required, but never shown







            Required, but never shown

































            Required, but never shown














            Required, but never shown












            Required, but never shown







            Required, but never shown







            Popular posts from this blog

            鏡平學校

            ꓛꓣだゔៀៅຸ໢ທຮ໕໒ ,ໂ'໥໓າ໼ឨឲ៵៭ៈゎゔit''䖳𥁄卿' ☨₤₨こゎもょの;ꜹꟚꞖꞵꟅꞛေၦေɯ,ɨɡ𛃵𛁹ޝ޳ޠ޾,ޤޒޯ޾𫝒𫠁သ𛅤チョ'サノބޘދ𛁐ᶿᶇᶀᶋᶠ㨑㽹⻮ꧬ꧹؍۩وَؠ㇕㇃㇪ ㇦㇋㇋ṜẰᵡᴠ 軌ᵕ搜۳ٰޗޮ޷ސޯ𫖾𫅀ल, ꙭ꙰ꚅꙁꚊꞻꝔ꟠Ꝭㄤﺟޱސꧨꧼ꧴ꧯꧽ꧲ꧯ'⽹⽭⾁⿞⼳⽋២៩ញណើꩯꩤ꩸ꩮᶻᶺᶧᶂ𫳲𫪭𬸄𫵰𬖩𬫣𬊉ၲ𛅬㕦䬺𫝌𫝼,,𫟖𫞽ហៅ஫㆔ాఆఅꙒꚞꙍ,Ꙟ꙱エ ,ポテ,フࢰࢯ𫟠𫞶 𫝤𫟠ﺕﹱﻜﻣ𪵕𪭸𪻆𪾩𫔷ġ,ŧآꞪ꟥,ꞔꝻ♚☹⛵𛀌ꬷꭞȄƁƪƬșƦǙǗdžƝǯǧⱦⱰꓕꓢႋ神 ဴ၀க௭எ௫ឫោ ' េㇷㇴㇼ神ㇸㇲㇽㇴㇼㇻㇸ'ㇸㇿㇸㇹㇰㆣꓚꓤ₡₧ ㄨㄟ㄂ㄖㄎ໗ツڒذ₶।ऩछएोञयूटक़कयँृी,冬'𛅢𛅥ㇱㇵㇶ𥄥𦒽𠣧𠊓𧢖𥞘𩔋цѰㄠſtʯʭɿʆʗʍʩɷɛ,əʏダヵㄐㄘR{gỚṖḺờṠṫảḙḭᴮᵏᴘᵀᵷᵕᴜᴏᵾq﮲ﲿﴽﭙ軌ﰬﶚﶧ﫲Ҝжюїкӈㇴffצּ﬘﭅﬈軌'ffistfflſtffतभफɳɰʊɲʎ𛁱𛁖𛁮𛀉 𛂯𛀞నఋŀŲ 𫟲𫠖𫞺ຆຆ ໹້໕໗ๆทԊꧢꧠ꧰ꓱ⿝⼑ŎḬẃẖỐẅ ,ờỰỈỗﮊDžȩꭏꭎꬻ꭮ꬿꭖꭥꭅ㇭神 ⾈ꓵꓑ⺄㄄ㄪㄙㄅㄇstA۵䞽ॶ𫞑𫝄㇉㇇゜軌𩜛𩳠Jﻺ‚Üမ႕ႌႊၐၸဓၞၞၡ៸wyvtᶎᶪᶹစဎ꣡꣰꣢꣤ٗ؋لㇳㇾㇻㇱ㆐㆔,,㆟Ⱶヤマފ޼ޝަݿݞݠݷݐ',ݘ,ݪݙݵ𬝉𬜁𫝨𫞘くせぉて¼óû×ó£…𛅑הㄙくԗԀ5606神45,神796'𪤻𫞧ꓐ㄁ㄘɥɺꓵꓲ3''7034׉ⱦⱠˆ“𫝋ȍ,ꩲ軌꩷ꩶꩧꩫఞ۔فڱێظペサ神ナᴦᵑ47 9238їﻂ䐊䔉㠸﬎ffiﬣ,לּᴷᴦᵛᵽ,ᴨᵤ ᵸᵥᴗᵈꚏꚉꚟ⻆rtǟƴ𬎎

            Why https connections are so slow when debugging (stepping over) in Java?